Applying the definition of a limit

  • Thread starter Thread starter sinbad30
  • Start date Start date
  • Tags Tags
    Definition Limit
Click For Summary
The discussion focuses on applying the definition of a limit to demonstrate that the limit of the function (x^3 * y(y-1)) / (x^2 + (y-1)^2) approaches 0 as (x,y) approaches (0,1). The participants analyze the inequalities involving |x| and |y-1|, using the triangle inequality to establish bounds. They introduce epsilon-delta arguments, aiming to find a delta that ensures the function's value is less than epsilon when the distance from (0,1) is sufficiently small. A key insight is recognizing that the distance r from (x,y) to (0,1) simplifies the right-hand side of the inequality to r^2(r+1). The conversation emphasizes the importance of translating the problem into manageable terms to facilitate the limit proof.
sinbad30
Messages
3
Reaction score
0

Homework Statement



Applying the definition of a limit to show that

lim ((x^3 * y(y-1) ) / (x^2 + (y-1)^2) = 0 as (x,y) approaches (0,1)



The Attempt at a Solution



|x| = sqrt(x^2) <= sqrt((x^2 + (y-1)^2))
|y-1|=sqrt((y-1)^2)<= sqrt((x^2 + (y-1)^2))
|y|<= |y-1| + 1 via the triangle inequality

Let e>0. We want to find d>0 such that

0 < sqrt((x^2 + (y-1)^2)) < d then (|x|^3 |y| |y-1|)/ (x^2 + (y-1)^2) < e

So

(|x|^3 |y| |y-1|)/ (x^2 + (y-1)^2) <=
[(sqrt((x^2 + (y-1)^2)))^3 * sqrt((x^2 + (y-1)^2)) * (sqrt((x^2 + (y-1)^2)) + 1) ] / (x^2 + (y-1)^2)

Any ideas as to how to break down the right side of the inequality?
 
Physics news on Phys.org
sinbad30 said:

Homework Statement



Applying the definition of a limit to show that

lim ((x^3 * y(y-1) ) / (x^2 + (y-1)^2) = 0 as (x,y) approaches (0,1)



The Attempt at a Solution



|x| = sqrt(x^2) <= sqrt((x^2 + (y-1)^2))
|y-1|=sqrt((y-1)^2)<= sqrt((x^2 + (y-1)^2))
|y|<= |y-1| + 1 via the triangle inequality

Let e>0. We want to find d>0 such that

0 < sqrt((x^2 + (y-1)^2)) < d then (|x|^3 |y| |y-1|)/ (x^2 + (y-1)^2) < e

So

(|x|^3 |y| |y-1|)/ (x^2 + (y-1)^2) <=
[(sqrt((x^2 + (y-1)^2)))^3 * sqrt((x^2 + (y-1)^2)) * (sqrt((x^2 + (y-1)^2)) + 1) ] / (x^2 + (y-1)^2)

Any ideas as to how to break down the right side of the inequality?
It's helpful to recognize that ##r = \sqrt{x^2 + (y-1)^2}## is the distance of the point (x,y) from the point (0,1). In terms of r, the righthand side is ##\frac{r^4(r+1)}{r^2} = r^2(r+1)##. Can you take it from there?
 
Question: A clock's minute hand has length 4 and its hour hand has length 3. What is the distance between the tips at the moment when it is increasing most rapidly?(Putnam Exam Question) Answer: Making assumption that both the hands moves at constant angular velocities, the answer is ## \sqrt{7} .## But don't you think this assumption is somewhat doubtful and wrong?

Similar threads

Replies
4
Views
2K
  • · Replies 14 ·
Replies
14
Views
2K
  • · Replies 4 ·
Replies
4
Views
2K
Replies
4
Views
2K
  • · Replies 5 ·
Replies
5
Views
1K
  • · Replies 10 ·
Replies
10
Views
2K
  • · Replies 10 ·
Replies
10
Views
2K
  • · Replies 10 ·
Replies
10
Views
2K
  • · Replies 21 ·
Replies
21
Views
2K
  • · Replies 2 ·
Replies
2
Views
2K